Three polarizers are more transparent than two

Поделиться
HTML-код
  • Опубликовано: 11 июл 2021
  • Two polarizers rotated at 90° to each other completely block light. But if you insert between them the third polarizer at 45° to both of them, such a system will partially transmit light.
    support@getaclass.ru
    #VeritasiumContest

Комментарии • 218

  • @repka4
    @repka4 3 года назад +55

    сделайте пожалуйста более подробный ролик на эту тему

  • @huston_verner
    @huston_verner 3 года назад +25

    Как вы победить то хотите!
    С каждым роликом всё лучше и лучше! Так держать!
    Недавно про парадокс Фарадея узнал, но на вашем канале не нашёл, было бы интересно, но это, видимо, после конкурса будет.

  • @user-fq1ee9vv3q
    @user-fq1ee9vv3q 3 года назад +4

    Никогда так ни о чём не сожалею, как о том, что пропустил "мимо ушей" электродинамику и оптику.
    Спасибо!!!
    Ещё в одиннадцатом классе, наш Николай Андреевич, нам это показал и объяснил ( с формулами)! А я имел наглость - забыть!?!
    Ещё раз - СПАСИБО!!!

  • @Uni-Coder
    @Uni-Coder 3 года назад +14

    Договоримся самый первый поляризатор не учитывать, т.к. он нужен только для того, чтобы сделать поляризованный свет, поскольку солнечный свет не поляризованный (если бы у нас был лазер, этот поляризатор нам бы и не понадобился).
    Далее, поляризованный свет проходит через поляризующую пластину с интенсивностью, равной cos alpha, где alpha - угол между направлениями поляризации света и пластины. По сути, это проекция вектора электрического поля на направление поляризации пластины.
    Соответственно через первый поляризатор (это как бы "второй", но мы самый первый договорились не учитывать) свет пройдёт с интенсивностью cos 45 = корень(2)/2 и сменит направление поляризации. Через второй поляризатор также пройдёт с интенсивностью корень(2)/2. Итого получим 1/2.
    Если же поляризатор только один, то свет проходит с интенсивностью cos90 = 0.
    Более того, можно увеличить количество поляризаторов. Например, если поставить 3 штуки, каждый под углом 30 градусов к предыдущему, мы получим корень(3)/2 три раза, и результирующая интенсивность будет около 65%. А если поставить 6 штук под углом 15, получим более 81%. Увеличивая их количество, можно добиться поворота направления поляризации на 90 градусов практически без потерь. Общая формула: cos(90/n)^n.
    Правда, это в идеальном случае... Реальные поляризаторы, конечно, имеют потери.
    Вкратце, в данном эксперименте речь о том, что комбинация двух проекций по 45 градусов не обязательно даёт тот же эффект, как одна проекция на 90 градусов. Математика: cos(2*alpha) не обязательно равно 2*cos(alpha).

    • @TENKARAru
      @TENKARAru 3 года назад +2

      Т.е.каждый поляризатор под углом около 45 гр.последовательно доворачивает поляризацию волны?

    • @Uni-Coder
      @Uni-Coder 3 года назад +1

      @@TENKARAru После любого поляризатора (неважно, какой угол) волна становится поляризованной в том направлении, в котором ориентирован сам поляризатор.
      От угла зависит интенсивность прошедшего света, она соответствует квадрату косинуса угла. Между направлением поляризации падающего света и поляризатора. (закон Малюса). Т.е. если угол 90 градусов, ничего не проходит, если угол 0 градусов, проходит всё, а если 45 градусов, проходит половина

    • @Uni-Coder
      @Uni-Coder 3 года назад +2

      Я написал "косинус", но по закону Малюса должен быть квадрат косинуса. Надо расчёты исправить, прошу прощения за возможные ошибки.

    • @TheStage10
      @TheStage10 2 года назад

      Пока читал ваш коментарий, понял что это же и есть принцип работы жк панелей, просто в каество второго (или "первого", если первый ре учитывать) - жидкие кристалы

    • @lansalvares1553
      @lansalvares1553 Год назад +1

      @@Uni-Coderт.е повернув второй поляризатор на 90 мы полностью блокируем свет. Но если повернуть второй на 45 градусов то из 100% света мы получаем 50%. А если добавить ещё третий поляризатор повёрнутый на 45 относительно второго, то эти 50% мы делим ещё на два. И остаётся 25% от начального света. Я правильно все понял?)

  • @MateusHokari
    @MateusHokari 3 года назад +4

    This geometrical explanation is really good! Good luck on your contest

  • @Skynet_is_Watching_You
    @Skynet_is_Watching_You 3 года назад +4

    Гениально, спасибо что рассказали

  • @Enot_-ik
    @Enot_-ik 3 года назад +2

    Продолжайте и не бросайте снимать зашла на ютуб и тут топ 5 лучших блогеров вот вас нашла подпишусь и повторяю не бросайте снимать как другие блогеры

  • @RobotN001
    @RobotN001 3 года назад +2

    Вот это видео весьма хорошо спутанно с опытом и экспериментом. 👍

  • @Nidvoraich
    @Nidvoraich 3 года назад +9

    Очень непонятное объяснение

  • @user-jh6yz1by9t
    @user-jh6yz1by9t 3 года назад +5

    поставьте между двумя поляризаторами кювету с раствором сахара . замерьте угол вращения плоскости поляризации света , зависимость этого угла от концентрации вещества покажите .

  • @DANJ-cg2gp
    @DANJ-cg2gp 3 года назад +2

    Ваш канал надо рекомендовать учителям физики в школах!

    • @user-sf5lh7ot5r
      @user-sf5lh7ot5r 3 года назад +2

      а мы и так тут сидим...

    • @DANJ-cg2gp
      @DANJ-cg2gp 3 года назад +1

      @@user-sf5lh7ot5r Кто-то сидит, а кто-то и не догадывается о его существовании!

  • @Евгений_Матвев
    @Евгений_Матвев 3 года назад +3

    Дерек вам наверное все призовые места отдаст

  • @777kasan
    @777kasan 3 года назад +1

    Вот это дааа!
    Вот это тема! 👍

  • @inelectronic
    @inelectronic 3 года назад

    Комментарий в поддержку канала)

  • @CastBulat
    @CastBulat 3 года назад

    Классно, даже не думал что такое может быть

  • @YuriyEliseev
    @YuriyEliseev 3 года назад +3

    Непонятно в чём парадокс, надо объяснить почему полностью поглощается свет в дух поляризаторах, и как это решается добавлением третьего

  • @user-vk1oh9pq2e
    @user-vk1oh9pq2e 3 года назад +2

    круто конечно ! я когда в школе учился, говорили что свет у фар автомашин можно поляризировать, а у водителей ставить козырек что бы отсекать этот свет. но как то не прижилось видимо.

  • @olgerdtmagpier5527
    @olgerdtmagpier5527 2 года назад

    Бомба!

  • @SuperVinigret
    @SuperVinigret 3 года назад

    коротко и ясно.

  • @mathempire314
    @mathempire314 3 года назад

    Интересный эксперимент

  • @gimeron-db
    @gimeron-db 3 года назад +5

    Ещё один вопрос. Допустим у нас есть много пластин, если промежуточные слои расставлять с небольшим поворотом нового слоя относительно предыдущего. На выходе мы получим такой же яркий свет, но уже с горизонтальной поляризацией?

    • @alexandersmolyakov1886
      @alexandersmolyakov1886 3 года назад

      На каждом поляризаторе проходит лишь часть света, поэтому даже если сделать поворот по 1 градусу, но пластин будет много, то и потерь будет много

    • @Uni-Coder
      @Uni-Coder 3 года назад +1

      @@alexandersmolyakov1886 В случае с идеальным поляризатором всё сработает совсем наоборот - много поляризаторов, расположенных через градус, на выходе дадут интенсивность, близкую к 100%. С реальными поляризаторами, конечно, всё утухнет. Но, я думаю, есть какая-то золотая середина, 4-5 поляризаторов (не знаю), которые дадут эффект больший, чем показано в видео.

    • @alexandersmolyakov1886
      @alexandersmolyakov1886 3 года назад

      @@Uni-Coder c идеальным поляризатором - согласен, но идеальные бывают только в теории

    • @Uni-Coder
      @Uni-Coder 3 года назад +2

      @@alexandersmolyakov1886 Вот и отличная задача. Положим, поляризатор пропускает только k-ю часть излучения независимо от поляризации. То есть с учётом поляризации пропустит k*cos a, где a - угол между направлением поляризации света и направлением поляризатора. Нам нужно расположить n поляризаторов равномерно, чтобы в итоге получилось 90 градусов, т.е. каждый угол будет равен 90/n. Найти оптимальное n, при котором свет будет проходить больше всего. Фактически, найти максимум функции f(n) = (k*cos(90/n))^n

    • @gimeron-db
      @gimeron-db 3 года назад +1

      ​@@alexandersmolyakov1886 Про поглощение и отражение обратно известно. Я предположил, что слои абсолютно прозрачны. Как бы тогда повёл себя свет?
      Надо будет прогнать расчёты: Предел произведений Cos(Pi/4 / n) при n стремящемуся к бесконечности.
      upd: Предел Cos(Pi/4 /n)^n при n стремящемуся к бесконечности равен единице.
      n f(n)
      1 0,7071067811865476
      2 0,8535533905932737
      3 0,9012210650134381
      4 0,9253281139039617
      5 0,9399389255049855
      6 0,9497545811946625
      7 0,9568070852894337
      8 0,9621208304101866
      9 0,9662689721912053

  • @basiliustschelovetscheskij8689
    @basiliustschelovetscheskij8689 3 года назад +1

    Комментарий в поддержку канала
    Чем страшнее был огонь,
    Чем больше свирепствовала битва,
    Защищая свое знамя,
    Легионер наступал.
    И, не боясь ударов
    Воодушевленного врага,
    Он сумел умереть отважно
    И отвоевать знамя.

  • @user-dg8gf8uw6b
    @user-dg8gf8uw6b 3 года назад +2

    Ничего не понял...но очень круто 👍

  • @user-fi3cd5sf3x
    @user-fi3cd5sf3x 3 года назад

    О, квантовая механика началась

  • @chubrik2
    @chubrik2 3 года назад +1

    Я ждал лонгспич про квантовую суперпозицию :(

  • @Aleksey_Sh
    @Aleksey_Sh 3 года назад +2

    Откуда второй поляризатор берёт лучи для себя? Ведь там прошли только вертикальные. Хорошо бы подробнее рассказать, что такое поляризатор и как он работает?

  • @stas_lu
    @stas_lu 3 года назад +1

    предлагаю оставить это формат роликов и далее)

  • @denisgrey
    @denisgrey 3 года назад

    Поляризировали, поляризировали, да не до выполяризировали ;)

  • @AxolotlIsMyTotem
    @AxolotlIsMyTotem 3 года назад +2

    Ничего не понял.

  • @Fink_from_GR
    @Fink_from_GR 8 месяцев назад

    Вопрос. Если свет стал паляризованным его можно вернуть в обычное состояние ( через другой "фильтр") ?

  • @disconnect7
    @disconnect7 3 года назад +1

    А где добыть поляризационные плёнки? Хотелось бы и самому с ними поиграться, и может быть сделать регулируемые солнцезащитные очки.

    • @basiliustschelovetscheskij8689
      @basiliustschelovetscheskij8689 3 года назад

      Из поляризующих очков или фильтров вынь

    • @user-vi7sy2pd1o
      @user-vi7sy2pd1o 3 года назад

      В матрице от старого/разбитого ноута например. Можно и от нового взять, но выйдет дороговато:)

  • @vadimushkevich1283
    @vadimushkevich1283 Год назад +1

    С большим трудом отбил, отделил, отодрал, отскрёб остатки стекла от пластиковых пластин-поляризаторов жидкокристаллических дисплеев. Затем не с меньшим трудом очистил их от клея. Когда смотрю на экран работающего компьютерного через две скрещенные пластин-поляризаторов то свет не проходит. Стоит их поменять местами (ближняя от своего глаза станет дальше, а дальняя - ближе) то свет проходит, хотя они остались таким же образом скрещены. И для того, чтобы свет не проходил нужно довернуть пластину на 90 градусов. Также, ситуация меняется, если пластину перевернуть другой стороной к себе. В чём тут дело?

  • @user-sj5vf3fh7o
    @user-sj5vf3fh7o 3 года назад +1

    Друзья расскажите пожалуйста про разницу с точки зрения физики диаметра колес на велике, мото, авто. Что легде крутить, где ввше скорость и вообще КПД

    • @JohnSmith-sd4vh
      @JohnSmith-sd4vh 3 года назад +3

      Легче крутить колёса велосипеда. Так как машина и мотоцикл больше и тяжелее.
      Ваш КО.

    • @Gottom-
      @Gottom- 3 года назад

      Чем меньше радиус, тем легче крутить.

    • @Sarmintu
      @Sarmintu 3 года назад +1

      @@Gottom- это только если по твердому и ровному ехать.

    • @user-sj5vf3fh7o
      @user-sj5vf3fh7o 3 года назад +2

      @@JohnSmith-sd4vh MIT ждет вас!!! Вы чертов гений сэр!!

    • @ElimDax01
      @ElimDax01 3 года назад

      Тема напрямую связана с рычагами, советую про них всё узнать

  • @KOT040188
    @KOT040188 3 года назад +3

    Ещё со школы мучаюсь вопросом, что есть это ваша поляризация? Как её представить? Не могу её понять.

    • @peacedalubof
      @peacedalubof 3 года назад

      Как забор из прутьев.

    • @disconnect7
      @disconnect7 3 года назад

      Это свойство света проходить через поляризационный фильтр (или блокироваться им).

    • @KOT040188
      @KOT040188 3 года назад

      @@disconnect7 вот так гораздо понятнее стало (нет).

    • @Sarmintu
      @Sarmintu 3 года назад +1

      В Википедии очень хорошо расписано. ru.m.wikipedia.org/wiki/%D0%9F%D0%BE%D0%BB%D1%8F%D1%80%D0%B8%D0%B7%D0%B0%D1%86%D0%B8%D1%8F_%D0%B2%D0%BE%D0%BB%D0%BD

    • @nipowpow
      @nipowpow 3 года назад

      Учебниках физики за 7й класс весьма доходчиво объясняется это явление

  • @luckiller420
    @luckiller420 3 года назад +3

    Не совсем понял, а откуда берётся свет под 45 градусов, если он не мог пройти через 1 поляризатор

    • @gimeron-db
      @gimeron-db 3 года назад +5

      Под 45 градусов был повёрнут второй.
      Поляризаторы пропускают как бы проекцию поляризации приходящего света на свою плоскость поляризации. После прохождения света через второй поляризатор поляризация света теперь не вертикальная, а под 45 градусов. Третий со вторым тоже под 45, а значит также пропустит часть света, поляризация которого потом станет горизонтальной.

    • @Misha-775
      @Misha-775 3 года назад +3

      Там такая штука, что поворотом на некий угол определяется ВЕРОЯТНОСТЬ конкретного кванта света преодолеть поляризатор. А вернее, там формула P = cos²(угол). А если квантов много, то и какие-то пройдут, а какие-то нет. Вот и выходит, что после прохождения 3-х поляризаторов свет в 4 раза тусклее, ибо прошли лишь ¼.

  • @Anti_During
    @Anti_During 2 года назад

    я понимаю это с точки зрения аналитики, через косинусы. или через геометрию - по проекциям. но как само явление осмыслить ))

  • @yatyomich9513
    @yatyomich9513 3 года назад +2

    А где можно купить такие поляризаторы? Или как правильно называется это изделие?

    • @sniker9671
      @sniker9671 3 года назад +1

      Их можно достать из жк экранов, например, из неработающего мультиметра или чего-то подобного

    • @nnmsaiu
      @nnmsaiu 3 года назад +1

      Из ЖК экрана можно вытащить.

    • @-igor-
      @-igor- 3 года назад +2

      @@sniker9671 А можно достать и из работающего :)

    • @TheBurzhuy
      @TheBurzhuy 3 года назад +2

      На али мне кажется должны быть, поляризационные фильтры это называется. есть еще в виде стекл для фотографов такие но там цены могут сильно кусаться

    • @xy-box
      @xy-box 3 года назад +1

      Самое интересное, что работоспособность он особо не потеряет

  • @Bilalline
    @Bilalline 3 года назад

    Вращатель поляризации

  • @drklmn7416
    @drklmn7416 Год назад +1

    Но это вообще ничего не объясняет. Почему порядок наложения фильтров меняет результат?

  • @user-vi7sy2pd1o
    @user-vi7sy2pd1o 3 года назад +12

    Тут квантмех замешан, я так полагаю.

    • @RobotN001
      @RobotN001 3 года назад +1

      и нелокальность естества o_O

    • @aliaruzstanbekov123isalego
      @aliaruzstanbekov123isalego 3 года назад

      Арриить тьтиии тооим мтимм иттипмпт

    • @user-xt9op7bu1j
      @user-xt9op7bu1j 3 года назад

      @@RobotN001 Ага сразу гравитационный парадокс

    • @Uni-Coder
      @Uni-Coder 3 года назад

      Просто комбинация двух проекций по 45 градусов не обязательно даёт тот же эффект, как одна проекция на 90 градусов. Математика: cos(2*alpha) не обязательно равно 2*cos(alpha). Максвелла тут вполне хватит, квантовую теорию поля можно не беспокоить.

    • @user-vi7sy2pd1o
      @user-vi7sy2pd1o 3 года назад

      @@Uni-Coder ну точно. Я тут сижу интегралы считаю, а оказывается всё просто...

  • @DVkot
    @DVkot 3 года назад +4

    И почему часть всё таки проходит, когда проходит половина, не?

  • @user-gr5qb2iv8w
    @user-gr5qb2iv8w 3 года назад +14

    объяснение не совсем корректное, так как полностью природу этого можно объяснить только при помощи квантовой механики.

    • @armand-jeanduplessis9777
      @armand-jeanduplessis9777 3 года назад

      Ох уж этот Шрёдингер!

    • @user-gr5qb2iv8w
      @user-gr5qb2iv8w 3 года назад

      @@armand-jeanduplessis9777 это к чему?

    • @obpaTu_BHuMaHue
      @obpaTu_BHuMaHue 3 года назад +2

      Блин куда же проще то.. :) а вы хотите усложнить.. хочется сложнее читай научную литературу :) а это канал для подростков :)

    • @Alex-5231-qt
      @Alex-5231-qt 3 года назад

      @@armand-jeanduplessis9777 Шрёдингер мошенник, он от расходящегося ряда в своей формуле отбросил хвост с бесконечностью.

    • @obpaTu_BHuMaHue
      @obpaTu_BHuMaHue 3 года назад

      @@zchz2224 ну ... Есть немного.. но не прям уж сложно..

  • @mike-stpr
    @mike-stpr 3 года назад

    Сколько нужно поляризаторов, чтобы заменить двоих? Ответ на этот вопрос отделит вдумчивых школьников от отличников-зубрил. СПАСИБО!!!!!!!!!!!!!!!!!!!!!!!!!!!!!!!!!!!!!!!!!!!!!!!!!!!!!!!!!!!!!!

  • @brachka77
    @brachka77 3 года назад

    Почему этого не было на уроках физики.. :'(

  • @AleksandrMyz
    @AleksandrMyz 2 года назад

    Если понять, как то, что уровень (яркость) проходящего света через поляризаторы относительно источника освещения и измерителя (камеры) с тремя поляризаторами выше, чем с двумя, то заголовок видео похож на кликбейт) Был ли он или такая формулировка названия видео оказалась наиболее подходящей?
    А по сути самого эксперимента все верно оказалось. Хоть и не очень понял сходу, нужно глубже изучать суть явления.

  • @MrGrefQn
    @MrGrefQn 3 года назад

    Не понял)

  • @Arhi78
    @Arhi78 3 года назад +3

    А где же "фром май харт?")

  • @basiliustschelovetscheskij8689
    @basiliustschelovetscheskij8689 3 года назад

    13 комментов, как-то не красиво

  • @ENROF
    @ENROF 3 года назад

    Колдовство

  • @postoronny
    @postoronny 3 года назад

    Хулиганы :)

  • @DVkot
    @DVkot 3 года назад +2

    Так почему светлее?)

    • @Alex-5231-qt
      @Alex-5231-qt 3 года назад +1

      А может ты шпиён. Мы ничего тебе не скажем!

  • @MyoriX23
    @MyoriX23 3 года назад

    Максимально непонятно

  • @rolocan_t
    @rolocan_t 3 года назад +6

    Ничего не понял....
    Какое-то объяснение не объяснение

    • @sillysad3198
      @sillysad3198 3 года назад

      da, eto video otstoj.
      na samom dele poljarizator povorchivajet poljarizaciju, poetomu i effekt.

    • @nikolaymatveychuk6145
      @nikolaymatveychuk6145 3 года назад +2

      посмотрите моё объяснение в комментариях, может что-то прояснит, хотя я тоже не мастак объяснять :) Я тоже считаю, что объяснение в видео было похоже на "вот так это работает, примите это и не спрашивайте почему"

  • @DVkot
    @DVkot 3 года назад

    Вас ребятки препода тролят) Через третий полярик светлее

  • @TheKelikat
    @TheKelikat 3 года назад

    нипанятна

  • @russru2502
    @russru2502 3 года назад +2

    Это говорит о возможности света огибать препятствие.
    Наверное.

  • @malishevg
    @malishevg 3 года назад

    До сих пор помню как учитель нам взорвал мозг неравенством Бэлла. Хотя, если честно, в minutephysics, объяснение поподробнее, также как и в оригинальном обсуждении (ещё до veritasium) про движение парома, поднимающегося против течения
    ruclips.net/video/zcqZHYo7ONs/видео.html

  • @sillysad3198
    @sillysad3198 3 года назад

    this is fail.

  • @WeekendRider100
    @WeekendRider100 3 года назад +1

    Тут вообще ничего странного, ведь пластинки то вообще прозрачные и блокируют только перпендикулярные лучи. Если взять десяток поляризаторов и разместить и под одним и тем же углом, то они ещё светлее будут.
    Лучше про движение по ветру быстрее ветра до конца объясните, а то слились, им типа всё понятно.

  • @basiliustschelovetscheskij8689
    @basiliustschelovetscheskij8689 3 года назад

    Я оставляю здесь своё (****) мнение, следовательно, я существую

  • @user-kt6xn1qn8q
    @user-kt6xn1qn8q 3 года назад +2

    Комментарий в поддержку канала)

    • @batcat1508
      @batcat1508 3 года назад +1

      коментарий к комментарию для поддержки канала